PT3.S2.Q4 - Please #help: the correct answer choice (B) is a mistaken reversal

lsatcucumberlsatcucumber Member
edited September 2022 in Logical Reasoning 11 karma

STATEMENT 1: "A work of architecture, if it is to be both inviting and **functional...
inviting + functional for public use --> ~obtrusive

contrapositive: obtrusive --> ~inviting or ~functional for public use

STATEMENT 2: "Modern architects, plagued by egoism...

We are thus told that modern architects (because they let their strong personalities take over their work) are producing buildings that are not functional for public use.

Answer choice B states that "modern architects who let their strong personalities take over their work produce buildings that are not unobtrusive." In other words, they produce obtrusive buildings.

The stimulus tells us that modern architects are producing buildings that are not functional for public use. We know from the contrapositive of the first statement that if a building is obtrusive, then it is either not inviting or it is not functional for public use. Does it follow that a building that is not functional for public use is obtrusive? No it does not: that would constitute a mistaken reversal of the conditional statement.

Please #help

Admin Note: Edited title. Please use the format: "PT#.S#.Q# - brief description of question." Also, removed the portions of the stimulus as posting the entire stimulus on the Forum is not allowed. See our Forum Rules here: https://7sage.com/discussion/#/discussion/15.

Comments

Sign In or Register to comment.